6 Exercise Six – Joinder and Supplemental Jurisdiction

I.  INTRODUCTION

A.  Joinder Devices Available under the Federal Rules

 

1.  Common Law and Code Practice

Joinder was harshly restricted under the common law.  Since a primary goal of common law pleading was reduction of the case to a single issue of law or fact [see Exercise Three, part I.A.3], the typical case was one plaintiff against one defendant on one theory of recovery.  Joinder of claims or parties on the grounds of convenience or judicial economy was not considered desirable.

Joinder of differing theories of recovery was difficult and often impossible because of the common law writ system.  Consider, for example, a plaintiff who wished to plead that defendant had both restrained him and announced to bystanders that he had caught a horse thief.  Could plaintiff join these two theories in a single action?  Since false imprisonment was properly brought under a writ of trespass, and slander was properly brought under a writ of case, the defendant could successfully demur for improper joinder.  The same result followed should plaintiff wish to plead defendant had taken a horse (trespass) and refused to return it (case).

Similarly, joinder of parties was difficult because plaintiff was the master of his own case.  While joinder of parties was possible, it depended on the substantive rights of the parties, which in turn depended on complicated rules that traced once again into the writ system.

Procedure in the court of equity was more flexible.  Equity did not attempt to hold a case to a single issue.  In fact, equity attempted to resolve an entire dispute in a single lawsuit; a popular equity maxim was “equity delights to do justice and not by halves.”  Accordingly, equity allowed plaintiffs to join various theories of recovery in a single suit, and equity allowed joinder of additional parties whose interests were implicated by the main controversy.

The middle 1800s brought on the wave of procedural reform of the codes.  The codes borrowed from both common law and equity, and adopted several joinder concepts from the latter.  See Charles E. Clark, Code Pleading chs. 6-7 (2d ed. 1947).  Joinder became easier.  Problems remained.  Joinder of theories of recovery was limited by narrow court interpretations of “transaction or transactions”[i] and also by the typical code provision that causes of action joined “must affect all the parties to the action.”  Joinder of parties was limited by narrow court interpretations of typical code provisions, such as that plaintiffs could join when they had “an interest in the subject of the action and in obtaining the relief demanded,” or that defendants could be joined when they “claim an interest in the controversy adverse to the plaintiff.”  See Charles E. Clark, Code Pleading 365, 382 (2d ed. 1947).  A case, decided as late as 1925, illustrates these difficulties.  In Ader v. Blau, 241 N.Y. 7, 148 N.E. 771 (1925), plaintiff sued two defendants for wrongful death.  The cause of action for negligence against the first defendant was for maintaining an attractive nuisance on which a boy hurt himself; the cause of action against the second defendant was for medical malpractice in treating the injuries.  The decision was that parties and causes of action were misjoined since the causes of action did not arise out of the same transaction and they did not affect all the parties.

2.  Federal Rules Joinder

The Federal Rules of Civil Procedure, promulgated in 1938, adopted the reforms of the codes and went further.  Because the drafters were able to identify the problem areas that had developed in joinder under the codes, they were able to eliminate many of the problems.  Joinder of both theories of recovery and of parties became easier.  This increased ease of joinder was tongue-in-groove with the new role of pleading.  Pleading under the common law and the codes served four functions: 1) giving the opponent and the judge notice of the nature of the claim (or defense), 2) weeding out groundless claims (or defenses), 3) revealing the facts of the case, and 4) narrowing the issues.  See Exercise Three, part I.A.3.  Pleading under the Federal Rules was designed primarily to give the opponent notice and to leave the functions of revealing facts, narrowing issues, and weeding out meritless claims to discovery.  See Exercise Three, part I.C.2.  Similarly, joinder was designed to promote broad  convenience and judicial economy, and to leave problems of confusion that such joinder might create to devices such as separation and severance for trial.  See Fed. R. Civ. P. 42.

A panoply of joinder devices is available under the Federal Rules.  In fact, the number of devices and the similarity of some of the names of the devices can cause confusion.  As a consequence, a person studying federal joinder must be sure to keep the various devices separate.  The task may seem daunting at first, but understanding the purpose of each joinder device should greatly reduce the difficulty.  This section provides the names and brief descriptions of all of the devices.[ii]  The next section of this exercise discusses each device in more detail.  The Federal Rules joinder devices are the following:

 

–joinder of claims

(party may join more than one claim against another party);

compulsory counterclaim (claim against an opposing party that arises out of the same transaction or occurrence as the claim of the opposing party);

permissive counterclaim (claim against an opposing party that does not arise out of the same transaction or occurrence);

cross-claim (claim against a co-party);

permissive joinder of parties (allows joinder of multiple plaintiffs or multiple defendants);

compulsory joinder of parties (requires joinder of multiple plaintiffs or multiple defendants);

third-party practice, or impleader (party defending a claim may bring into the action a third person who may be derivatively liable for all or part of the claim);

intervention of right (third person must be allowed to enter the action as a party);

permissive intervention (third person may be allowed to enter the action as a party); and

interpleader (person holding property potentially subject to multiple claimants may require claimants to assert their claims against the property in the same action–can be statutory interpleader or interpleader under the rule).

B.  Supplemental Jurisdiction

 

1.  The History of Pendent and Ancillary Jurisdiction.

As part I.A discussed, the passage of years has brought more generous joinder.  With this loosening of restrictions on the joinder devices themselves, attention in the field of joinder–at least in the federal courts–has shifted from the joinder devices to questions of federal subject matter jurisdiction.[iii]  For example, a plaintiff may be allowed as a matter of joinder of claims to attach a state law claim to a federal question claim, but why is a federal court allowed to adjudicate the state law claim?  Or a defendant may be allowed as a matter of joinder to assert a state law cross-claim against a co-defendant, but why is a federal court allowed to adjudicate the state law claim?

The federal courts created two common law doctrines that expanded their jurisdictional reach over joined claims and parties: pendent jurisdiction and ancillary jurisdiction.  Both of these common law doctrines have been subsumed into the statutory doctrine of supplemental jurisdiction.

Pendent jurisdiction allowed a plaintiff who asserted a federal question claim to add on, or append, additional state law theories of recovery arising out of the same facts as the federal claim.  Assume, for example, plaintiff had been fired by her employer, and wished to assert three theories of recovery against defendant: a civil rights violation under Title VII, a breach of contract under state law, and the tort of intentional infliction of emotional distress under state law.  This three-count complaint could be filed in state court.  Could all three counts be brought into federal court?  Yes, said pendent jurisdiction.  Since the two state-law theories arose out of a “common nucleus of operative fact,” all formed part of the same “case” under Article III and the entire case could be heard in federal court.[iv]

The pendent state law theories were limited to those factually intertwined with the federal law theory.  In the example above, plaintiff could bring a three-count complaint for 1) Title VII violation, 2) breach of employment contract, and 3) intentional infliction of emotional distress.  Pendent jurisdiction would not allow 4) a factually-unrelated traffic accident between plaintiff and one of defendant’s trucks.  On the other hand, pendent jurisdiction would likely allow 4) defamation for a reference letter sent to one of plaintiff’s prospective new employers.

Attempts were made to expand pendent jurisdiction to cover pendent parties as well as pendent “claims.”  These attempts to create pendent party jurisdiction achieved some success in lower federal courts, but were repeatedly rejected in the Supreme Court.[v]

While pendent jurisdiction assisted plaintiffs, ancillary jurisdiction assisted defendants.  A defendant properly brought into federal court was allowed by the doctrine of ancillary jurisdiction to assert any claims it had that arose out of the same transaction or occurrence as the original, jurisdictionally-proper claim.  Thus, for example, a defendant was allowed to assert a compulsory counterclaim against the plaintiff because a compulsory counterclaim by definition arose out of the same transaction or occurrence as the claim.

Operation of ancillary jurisdiction in most cases became quite mechanical.  The test for ancillary jurisdiction was whether the joined claim arose out of the same transaction or occurrence.  The same test was found in many of the joinder devices.  Therefore, when the joinder device was satisfied, ancillary jurisdiction was satisfied.  Ancillary jurisdiction covered compulsory counterclaims, cross-claims, third-party claims, and intervention of right; it did not cover permissive counterclaims and permissive intervention.

The concept of the same transaction or occurrence in its essence means a single set of facts, so the kinship of ancillary jurisdiction to pendent jurisdiction’s “common nucleus of operative fact” is readily apparent.  Therefore, merger of the two doctrines became sensible.

 

2.  Supplemental Jurisdiction

Congress decided in 1990 to merge the two jurisdictional doctrines by statute, and to call the result “supplemental jurisdiction”:

(a) Except as provided in subsections (b) and (c) or as expressly provided otherwise by Federal statute, in any civil action of which the district courts have original jurisdiction, the district courts shall have supplemental jurisdiction over all other claims that are so related to claims in the action within such original jurisdiction that they form part of the same case or controversy under Article III of the United States Constitution.  Such supplemental jurisdiction shall include claims that involve the joinder or intervention of additional parties.

(b) In any civil action of which the district courts have original jurisdiction founded solely on section 1332 of this title [diversity jurisdiction], the district courts shall not have supplemental jurisdiction under subsection (a) over claims by plaintiffs against persons made parties under Rule 14, 19, 20, or 24 of the Federal Rules of Civil Procedure, or over claims by persons proposed to be joined as plaintiffs under Rule 19 of such rules, or seeking to intervene as plaintiffs under rule 24 of such rules, when exercising supplemental jurisdiction over such claims would be inconsistent with the jurisdictional requirements of section 1332.[vi]

In later sections of this exercise, we will explore this statute in depth and apply it in a wide variety of joinder situations.  We will see that it is not a paragon of legislative drafting and has several unintended consequences.  For now, we note that in many ways § 1367 does exactly what it was intended to do.  It brings together pendent jurisdiction and ancillary jurisdiction into the new doctrine of supplemental jurisdiction.  It establishes the test for supplemental jurisdiction as “the same case or controversy under Article III,” which is largely synonymous with the common nucleus of operative fact and the transaction or occurrence.  It creates pendent party jurisdiction in the last sentence of paragraph (a).  It restricts the use of supplemental jurisdiction in diversity cases in paragraph (b) so as not to tread on the doctrine of complete diversity.

      Consequently, any joinder question requires a series of analytical steps.  The first step is to determine whether the joinder device permits the joinder.  The second step is to determine whether the federal court has independent subject matter jurisdiction over the claim or party to be joined.  When the federal court has independent jurisdiction, this second step is satisfied.  When, however, the federal court does not have independent jurisdiction over the added claim or party, then the supplemental jurisdiction statute must be consulted.  The third step is to determine that the claim or party to be added is part of the same case or controversy under Article III as required by § 1367(a).  If so, and federal jurisdiction is not based solely on diversity, the analysis is at an end.  If the federal jurisdictional basis is diversity, then a fourth step is required.  The fourth step demands careful parsing of § 1367(b) to make sure that it does not take away the federal supplemental jurisdiction that § 1367(a) granted.  As can be seen, only the first step involves operation of the actual joinder device.

II.  THE JOINDER DEVICES

A.  Claims

 

1.  Joinder of Claims

As stated previously, the common law sharply restricted joinder of claims in its search for a single issue in an action.  The codes broadened claim joinder by enumerating several possibilities for joinder, but the courts became caught up in technical and narrow definitions of the terms in those statutes.

The Federal Rules removed any possible questions about joinder of claims.  Joinder of claims under Federal Rule 18 is unrestricted: “A party asserting a claim, counterclaim, crossclaim, or third-party claim may join, as independent or alternate claims, as many claims as it has against an opposing party.”  The overriding policy is efficiency, allowing both the court and the parties to resolve all disputes in a single lawsuit.  That means a plaintiff who has a conglomeration of totally unrelated claims against a defendant may join them all in one action–although Federal Rule 10 suggests strongly that unrelated claims be stated in separate counts.  It means also that other parties who properly bring a transactionally-related claim, such as a counterclaim or a cross-claim, are able to add unrelated claims.

Should a confusing mess result, the solution of the rules is to allow the court to use its discretion under Federal Rule 42(b) to order separate trials.  Joinder of claims is not a pleading problem; it is a trial problem.

Of course the rule makes clear that claims “may” be joined.  A party may choose either to add another claim, or to save it for a later lawsuit.  A party choosing the latter course must be wary of the preclusion doctrines.  Should the unasserted matter actually be part of the same claim asserted in the first suit, it would be lost under the doctrine of claim preclusion (res judicata).  Even should the unasserted matter truly be a separate claim, one or more common issues may be litigated and decided in the first lawsuit, raising the possibility of issue preclusion (collateral estoppel).  See Exercise Eleven.

2.  Supplemental jurisdiction

Pendent jurisdiction allowed a claimant asserting a federal question to add a state law theory of recovery when both arose from a “common nucleus of operative fact.”  The same result is allowed under supplemental jurisdiction since the common nucleus of operative facts is essentially equivalent to the “same case of controversy under Article III” required by § 1367(a).

What about the situation when plaintiff has two factually unrelated claims against defendant?  When federal jurisdiction is based on a federal question, the second, state law claim would not qualify for supplemental jurisdiction because–being factually unrelated–it would by definition not qualify as “part of the same case or controversy under Article III.”  When the federal basis is diversity, then supplemental jurisdiction is not necessary because diversity exists and plaintiff will likely be able to aggregate the amounts of all the claims.

B.  Counterclaims

 

A counterclaim is asserted against an “opposing party.”  Fed. R. Civ. P. 13(a)(1), (b).  Essentially that means a counterclaim crosses the “v” of the lawsuit.  A defendant may assert a counterclaim against plaintiff.  A plaintiff may assert a counterclaim against a counterclaiming defendant.  A third-party defendant may counterclaim against a third-party plaintiff.  A claim that does not cross the “v,” as for example a defendant against another defendant, is not a counterclaim.

The roots of counterclaim practice can be found in the common law.  The practices of recoupment and setoff were available, but each had limitations.  Recoupment by a defendant was limited to a claim arising from the same contract or transaction as the plaintiff’s claim.  As such, it could reduce or eliminate plaintiff’s recovery, but could not provide a positive recovery for defendant.  Setoff was created in equity to remedy these weaknesses of recoupment.  Setoff allowed a positive recovery, and the defendant’s claim did not have to arise from the same transaction as the plaintiff’s claim.  Yet setoff had its own weaknesses: the claim to be set off had to be liquidated or subject to ready computation.  The right of setoff had to be mutual.  Most importantly, since setoff was an equitable procedure, the circumstances had to call for the action of equity.  For example, when A and B owed each other money, and A sued B for the debt, B could use setoff only when A was insolvent; otherwise B could bring a separate action.

The codes invented the counterclaim.  Typically, they limited its use to situations where the counterclaim arose from the same transaction, or from the same contract, as the claim.

The Federal Rules create two types of counterclaims: compulsory counterclaims and permissive counterclaims.

1.  Compulsory counterclaims

A compulsory counterclaim is a counterclaim that arises out of the same transaction or occurrence as the claim.[vii]  Other counterclaims are permissive counterclaims.  Courts are therefore required to determine the scope of the “transaction or occurrence.”  While that phrase cannot be precisely defined, in its essence a transaction or occurrence is a single set of facts.  It is not tied to legal theories or defenses.  When cars driven by A and B collide, and then driver B gets out of his car and punches driver A, this presents one transaction or occurrence (only one event), not one for negligence and another for battery.  Clearly, the transaction or occurrence is close kin to the claim for relief [see Exercise Three, part I.C.2], to the common nucleus of operative fact of pendent jurisdiction, to the Article III case or controversy of supplemental jurisdiction, and to the scope of a claim for purposes of res judicata [see Exercise Eleven, part II.C].

Unfortunately, many federal courts have felt a need to gloss the rule.  The four popular glosses are 1) whether res judicata would apply to a second suit on defendant’s claim, 2) whether the issues of fact and law in the claim and counterclaim are largely the same, 3) whether the same evidence will support or defeat both the claim and the counterclaim, and 4) whether the claim and counterclaim have a logical relationship.  Why “transaction or occurrence” requires gloss is puzzling.  Courts that look to a single set of facts follow the language of the rule, not an unnecessary addition.  The logical relationship standard may make sense for the more difficult decision of when to tie transactions together into the same claim, as is sometimes necessary to decide for a question of joinder of parties [see part II.E.1, infra] or in the field of res judicata.[viii]  It should be unnecessary here.  See Douglas D. McFarland, In Search of the Transaction or Occurrence: Compulsory Counterclaims, 40 Creighton L. Rev. 699 (2007).

The party possessing a compulsory counterclaim “must state” it.  Even though a defending party is thus required to litigate its claim in a forum of the opposing party’s choosing, the drafters decided that this inconvenience was justified by the efficiency of litigating all claims arising from the same transaction or occurrence in one proceeding.  A compulsory counterclaim that is not stated is lost, although courts vary on the theory of loss, some using preclusion, others using an estoppel, and others using a sanction for violation of the rules.[ix]  Clearly, the safe course for an attorney in doubt as to whether a client’s potential counterclaim is compulsory or permissive is to plead it.

2.  Permissive counterclaims

The Federal Rules define a permissive counterclaim by exclusion.  A permissive counterclaim is any counterclaim that is not compulsory.  “A pleading may state as a counterclaim against an opposing party any claim that is not compulsory.”  Fed. R. Civ. P. 13(b).

As the name suggests, a party may assert the permissive counterclaim in the action or may instead sue on it in a separate action–at a time and place of the party’s choosing.  Since by definition the permissive counterclaim does not involve the same subject matter as the claim, little efficiency is lost.  A party choosing not to bring a permissive counterclaim must at the same time be careful that it is not lost through the operation of issue preclusion; to the extent that the counterclaim has an issue (or issues) in common with the claim, the decision on that issue in the litigation of the claim may well be preclusive in a later, separate action on the counterclaim.  See Exercise Eleven, part II.B.

3.  Supplemental Jurisdiction for Counterclaims

Prior to the enactment of supplemental jurisdiction, the law in the area was clear.  Compulsory counterclaims, arising out of the same transaction or occurrence, qualified for ancillary jurisdiction; permissive counterclaims, not arising out of the same transaction or occurrence, did not qualify for ancillary jurisdiction.

Enactment of § 1367 in 1990 was not intended to change, and did not change, these results.  Compulsory counterclaims ride into federal court on supplemental jurisdiction.  Permissive counterclaims do not.

Compulsory counterclaim.  Looking first to § 1367(a), the court must decide whether the counterclaim is part of the same case or controversy under Article III.  By definition, a compulsory counterclaim, because it must arise out of the same transaction or occurrence, is part of the same Article III case or controversy.  Looking next to § 1367(b), the court will recognize that in diversity cases, “claims by plaintiffs against persons made parties under Rule 14, 19, 20, or 24″ are not within supplemental jurisdiction.  A counterclaim is asserted under Rule 13.  Since that rule is not on the list, § 1367(b) does not apply, and the court is back to § 1367(a).  A compulsory counterclaim is carried into federal court by supplemental jurisdiction.

Permissive counterclaim.  Looking first to § 1367(a), the court must decide whether the counterclaim is part of the same case or controversy under Article III.  By definition, a permissive counterclaim, because it does not arise out of the same transaction or occurrence, is not part of the same Article III case or controversy.  The court need not even consider § 1367(b).

C.  Crossclaims

1.  Joinder of Crossclaims

The crossclaim traces back into the equity courts, which allowed a party to assert a cross-bill against another party.  This procedure found its way into the federal equity rules of 1912.  Many code states adopted the procedure, usually renaming the device a cross-complaint.  See Jack H. Friedenthal, Mary Kay Kane & Arthur R. Miller, Civil Procedure § 6.7 (4th ed. 2005).

The Federal Rules carried forward the possibility of asserting a claim against another party to the action, either as a counterclaim against an opposing party or as a crossclaim against a coparty.  A coparty is a party on the same side of the “v.”  In other words, a crossclaim is by a defendant against another defendant.  Or a crossclaim is by one plaintiff against another plaintiff.  For example, assume plaintiff A and plaintiff B sue defendant C and defendant D.  C could crossclaim against D since they are coparties. [Note that should C assert such a crossclaim, C and D would then become opposing parties, and a claim by D back against C would be a counterclaim.] Or C and D could assert a counterclaim against A and B.  A might then plead a crossclaim against B (perhaps for indemnity).

Federal Rule 13(g) governs crossclaims in federal practice: “A pleading may state as a crossclaim any claim by one party against a coparty if the claim arises out of the transaction or occurrence that is the subject matter either of the original action or of a counterclaim * * *.”  The rule accordingly makes four things clear.  First, a crossclaim is a claim against a coparty.  Second, a crossclaim is always permissive.  Third, the party must state a “claim;” an assertion that a coparty is entirely liable should be pleaded as a denial, not as a crossclaim.  Fourth, the crossclaim must arise “out of the transaction or occurrence” of the original claim or counterclaim.  The concept of transaction or occurrence means in its essence the same set of operative facts [see II.B.1, supra].

Allowing parties to add factually related claims to an existing action makes efficient sense for the court; allowing the addition of unrelated claims to an existing action would serve no efficiency purpose.  That is why a crossclaim must be part of the same transaction or occurrence.  This reasoning is undercut somewhat, however, by the fact that once a party is able to plead a crossclaim, the party is then able to add other, completely unrelated claims to the same action.  This is so because of the broad federal joinder of claims rule, which allows the joinder of all claims against a party.  See II.A.1, supra.

2.  Supplemental Jurisdiction for Crossclaims

Prior to the enactment of supplemental jurisdiction, crossclaims qualified for ancillary jurisdiction because by rule they are required to arise from the same transaction or occurrence as the claim, and the same transaction or occurrence was also the test for ancillary jurisdiction.  Enactment of § 1367 in 1990 was not intended to change, and did not change, this result.

As with any question of supplemental jurisdiction, we begin with § 1367(a).  It provides supplemental jurisdiction extends to “all other claims that are so related to claims in the action within such original jurisdiction that they form part of the same case or controversy under Article III * * *.”  A crossclaim, because it must arise from the same transaction or occurrence, is part of the same case or controversy.  Supplemental jurisdiction exists.  We then consult § 1367(b).  It provides supplemental jurisdiction does not exist in diversity cases when joinder is accomplished under certain enumerated rules.  Rule 13(g) is not on the list.  Consequently, crossclaims will always be covered by supplemental jurisdiction.

D.  Third-party claims [also known as Impleader]

1.  Joinder of Third-party Claims

Third-party practice is commonly called impleader, and the two terms are synonymous.  The only difficulty with use of the term impleader is that it is another joinder device beginning with “i,” and sometimes this causes confusion.  A person must remember that impleader is used by a party to bring a person not a party (a third party) into the action, intervention is used by a person not a party to the action to force his way into the action, and interpleader is used by a person subject to multiple claims to the same property to force all claimants to assert those claims in a single action.

Third-party practice finds its origins in a common law procedure called “vouching in,” or “vouching to warranty.”  This procedure allowed a defendant to vouch in another person who would be liable (originally because the third person had given a warranty on the property sought from defendant); this allowed the vouched in party to assume defense of the action.  A judgment against the original defendant would then also be conclusive on the vouched in party.  The weakness of this procedure was that the original defendant was still required to bring a second, separate action against the vouched in party to obtain a judgment.  Third-party practice was adopted by several of the code states, and subsequently by the Federal Rules.

The advantage of third-party practice lies in this example.  Plaintiff consumer sues defendant retailer for selling a defective product.  The retailer can defend the action, and–should it lose–later sue the manufacturer of the product in a separate action.  When the retailer wins that second action, the manufacturer ultimately pays the damages.  The retailer is removed from the middle.  Drawbacks exist with this plan, however.  First, inconsistent results might occur: the jury in the first action may decide the product was defective, and the jury in the second action may decide the product was not defective.  Second, delay results.  The retailer might have to pay the first judgment years before the second case proceeds to judgment.  Even worse, during the time lag the statute of limitations on the second action might expire.  Third, the retailer will incur the expense of litigating two separate actions.

Impleader removes these problems.  By impleading the manufacturer into the original action, the retailer removes the possibility of inconsistent results since the same jury will decide the entire action.  Judgment will be entered on both the original claim and the third-party claim at the same time, so no delay results.  Both claims will be determined in the same litigation, so little added expense will result.

Prior to the general, plainer English, re-writing of the Federal Rules in 2007, the third-party practice rule was one large, complicated paragraph.  The amendment broke the rule into several smaller, more understandable parts.  Even so, in order to assist understanding third-party practice, we parse out each sentence of the rule.  Here are the relevant portions of Federal Rule 14(a), interspersed with our comments in italics.

 

A defending party may, as third-party plaintiff, serve a summons and complaint on a nonparty who is or may be liable to it for all or part of the claim against it.  Fed. R. Civ. P. 14(a)(1) (in part).

 

Third-party practice, or impleader may be used only against “a nonparty.”  A claim against an opposing party is a counterclaim.  A claim against a co-party is a crossclaim.  When plaintiff sues defendant, and defendant brings in a third party, defendant then becomes known as “defendant and third-party plaintiff.”  The original plaintiff remains the plaintiff, and the nonparty brought into the action is the third-party defendant.

This language also contains the most important thing to remember about the joinder device: impleader liability must be derivative.  A third-party claim asserts the third-party defendant “is or may be liable” to the original defendant for the defendant’s liability to the plaintiff.  Impleader is not a device to offer up an alternative defendant to the plaintiff.  For example, plaintiff homeowner sues defendant waterproofing company because the basement continues to leak.  Defendant can implead the manufacturer of the waterproof paint it used.  That is derivative liability.  Defendant cannot implead the architect of the house on the theory that the fault lies in the house design instead of the waterproofing job.  That is an alternate defendant, not derivative liability.  Rule 14 cannot be used for that purpose.  Defendant should plead a denial.

 

But the third-party plaintiff must, by motion obtain the court’s leave if it files the third-party complaint more than 14 days after serving its original answer..  Fed. R. Civ. P. 14(a)(1) (in part).

 

The original defendant may implead as a matter of right within 14 days of serving the original answer (although another party may later move to strike the third-party complaint, so impleader in the end is always discretionary with the court); of course, the common practice is to serve the third-party complaint as part of the same document as the answer.  After the 14-day period has expired, defendant must obtain leave of court to use third-party practice.  The court will decide whether the increased efficiency of a single action will outweigh any prejudice to a party.

 

The person served with the summons and third-party complaint–the ‘third-party defendant’:

(A) must assert any defense against the third-party plaintiff’s claim under Rule 12;

(B) must assert any counterclaim against the third-party plaintiff under Rule 13(a), and may assert any counterclaim against the third-party plaintiff under Rule 13(b) or any crossclaim against another third-party defendant under Rule 13(g);

(C) may assert against the plaintiff any defense that the third-party plaintiff has to the plaintiff’s claim * * *..  Fed. R. Civ. P. 14(a)(2) (in part).

 

The third-party defendant can defend the third-party claim, counterclaim against the third-party plaintiff (original defendant), cross-claim against other third-party defendants (if any–this is unlikely), and defend–assist in defense of–the original claim.  After all, if the original claim fails, no liability will pass through.

 

[The third-party defendant] may also assert against the plaintiff any claim arising out of the transaction or occurrence that is the subject matter of the plaintiff’s claim against the third-party plaintiff..  Fed. R. Civ. P. 14(a)(2)(D).

 

The third-party defendant may assert a claim directly against the original plaintiff, but that claim must be related to the action or efficiency would not result; accordingly, the claim must arise out of the same transaction or occurrence as the plaintiff’s claim.  The proper title for such a claim is a Rule 14 claim; it is not a counterclaim since the third-party defendant and the plaintiff are not opposing parties until such a claim is asserted, and it is not a cross-claim since they are not co-parties.

 

The plaintiff may assert against the third-party defendant any claim arising out of the transaction or occurrence that is the subject matter of the plaintiff’s claim against the third-party plaintiff.  The third-party defendant must then assert any defense under Rule 12 and any counterclaim under Rule 13(a), and may also assert any counterclaim under Rule 13(b) or any crossclaim under Rule 13(g)..  Fed. R. Civ. P. 14(a)(3).

 

The original plaintiff is allowed to assert a claim directly against a third-party defendant who the original defendant has brought into the action, so long as the claim is transactionally related.  The third-party defendant is then allowed to defend the claim as would an original defendant.  Should a claim by the plaintiff against the third-party defendant be asserted first, it would be a Rule 14 claim; should it be asserted after the third-party defendant has asserted a claim directly against plaintiff, it would be a counterclaim since the two parties have become opposing parties.

 

Any party may move to strike the third-party claim, to sever it, or to try it separately..  Fed. R. Civ. P. 14(a)(4).

 

As mentioned above, the court will decide whether efficiency outweighs any possible prejudice to a party of trial in a single action.

 

A third-party defendant may proceed under this rule against a nonparty who is or may be liable to the third-party defendant for all or part of any claim against it..  Fed. R. Civ. P. 14(a)(5).

 

This would be properly termed a fourth-party action.  The third-party defendant would become the “third-party defendant and fourth-party plaintiff.”  The above portions of Rule 14 apply, just as they do to a third-party action.  And the chain of actions can, at least in theory, continue.

 

2.  Supplemental Jurisdiction for Third-party Claims

Prior to the enactment of supplemental jurisdiction, third-party claims qualified for ancillary jurisdiction (a derivative claim must arise from the same transaction or occurrence).  Enactment of § 1367 in 1990 was not intended to change, and did not change, this result.

In the standard third-party practice situation, defendant impleads the third-party defendant.  The statute provides that supplemental jurisdiction extends to “all other claims that are so related to claims in the action within such original jurisdiction that they form part of the same case or controversy under Article III * * *.”  28 U.S.C. § 1367(a).  A third-party claim, because it must arise derivatively through the original claim, is part of the same case or controversy.  Supplemental jurisdiction exists under § 1367(a).  We then consult § 1367(b).  It provides supplemental jurisdiction does not exist in diversity cases for joinder “over claims by plaintiffs against persons made parties under Rule 14 * * *.”  This sentence does not apply: even though joinder is accomplished under Rule 14, the third-party claim is not a claim by a plaintiff.  It is a claim by a defendant.  Consequently, supplemental jurisdiction exists over the third-party claim.

The same result was intended to apply, and does apply, in two other third-party practice situations.  When the third-party defendant brings in a fourth-party defendant, that also is not a claim by a plaintiff, so supplemental jurisdiction attaches.  Similarly, when the third-party defendant asserts a claim directly against the original plaintiff, that also is not a claim by a plaintiff, so supplemental jurisdiction applies.

The opposite result was intended to apply, and does apply, when the original plaintiff asserts a claim directly against the third-party defendant.  Even though this claim must arise from the same transaction or occurrence, so § 1367(a) is satisfied, it is a claim by a plaintiff against a person made party under Rule 14, so § 1367(b) eliminates the supplemental jurisdiction.  Owen Equip. & Erection Co. v. Kroger, 437 U.S. 365, 98 S. Ct. 2396, 57 L.Ed.2d 274 (1978), held that ancillary jurisdiction did not extend to such a claim.  The drafters intended to, and did, carry this result forward in § 1367.

E.  Joinder of Parties

 

1.  Permissive Joinder of Parties

The common law, in its search for a single issue, made joinder of parties difficult.  It tied joinder to the substantive rights of the parties and the forms of action.  It distinguished between joint interests in which joinder was possible and several interests in which joinder was not possible.  The codes allowed joinder more generously, although they added their own artificial categories for when joinder of parties would be permitted.  See generally Jack H. Friedenthal, Mary Kay Kane & Arthur R. Miller, Civil Procedure § 6.4 (4th ed. 2005).

While joinder of parties under Federal Rule 20 is not freely allowed as is joinder of claims under Federal Rule 18 [see II.A.1, supra], the two requirements of Federal Rule 20(a) for permissive joinder of parties are minimal:

–the relief sought arises from the “same transaction, occurrence, or series of transactions or occurrences,” and

–a common question of law or fact will arise.[x]

The transaction or occurrence test arises throughout the federal joinder devices   We have already discussed its meaning with regard to compulsory counterclaims, for example [see II.B.1, supra].  The essence of a transaction or occurrence is a single set of facts; it is not in any way tied to legal theories of recovery or defenses.  Federal Rule 20 goes even further than a single transaction or occurrence: it allows permissive joinder when the relief arises from a series of transactions or occurrences.  Perhaps, for example, plaintiff is injured in an auto accident, and several months later the physician treating her for the accident injuries commits malpractice.  Plaintiff can join the driver and the physician permissively as defendants since this is a series of transactions or occurrences–even though separated in time by several months.  Or perhaps a salesman of worthless securities sells them to plaintiff A over the telephone and some time later sells them to plaintiff B during an in-home presentation.  The court would likely determine this to be a series of transactions or occurrences so that the two buyers could join permissively as plaintiffs in a single action.  Here is where the logical relationship test, considering judicial economy and convenience, makes sense [see II.B.1, supra].

The common question requirement is even easier to satisfy.  The question may be law or fact.  In the first example above, the extent and valuation of plaintiff’s combined injuries would provide a common question of fact.  In the second example, the fraudulent nature of defendant’s securities sales would provide a common question of law.  The rule does not require a majority of common questions, or even a multitude of common questions.  It requires only a common question.  Once again, consideration of whether a common question is presented will prompt the court to consider economy and convenience of trying the case in a single proceeding.

Should the court determine that parties are misjoined, the remedy is to drop the misjoined party, not to dismiss the case.  See Fed. R. Civ. P. 21.

2.  Compulsory Joinder of Parties

The common law required joinder of parties in certain limited situations, chiefly when a joint interest was involved.  The codes generally carried this requirement forward.  In interpreting this requirement in Shields v. Barrow, 58 U.S. (17 How.) 130, 15 L.Ed. 158 (1855), the Supreme Court distinguished between merely necessary parties, without whom the action could proceed, and indispensable parties, without whom the action could not proceed.  A party was deemed indispensable when the action would affect the absent party’s interest or the action could not provide complete relief without the absent party.  Over the following years, courts tended to sidestep the facts of the individual case in their haste to apply one of these two conclusory labels.

When Federal Rule 19 was originally promulgated in 1938, it adopted this system.  The title of the rule was “Necessary Joinder of Parties,” and it referred to “persons having a joint interest” who “shall be made parties.”  The same difficulties of the code systems accompanied the new rule.  Courts had great difficulty distinguishing between “indispensable” and “necessary” parties, and tended to slap conclusory labels on them.  This resulted in complete rewriting of Federal Rule 19 in 1966.

The title of Federal Rule 19 is now “Required Joinder of Parties.”  It attempts to avoid the labels of necessary and indispensable parties and directs the court faced with a question of whether a third party must be joined in an action to make a pragmatic decision based on the individual case.

First, a court must consider Rule 19(a) to determine whether the person is “to be joined if feasible.”  The inquiry is designed to investigate how strong the third party’s interest is in the case.[xi]  The rule provides guidelines to the court, primarily considering whether the absent person’s interest will be affected or whether complete relief is possible without the absent person.  Should the court decide the person is one who “must be joined,” the court “must order that the person be made a party.”

In the event joinder of the person is not feasible (joinder would destroy diversity or the person is not subject to personal jurisdiction), then the court must proceed to Rule 19(b).  That rule leads the court through consideration of four practical factors to determine whether the better course is to proceed without the absent person or to dismiss the action.  The rule requires the court to place the conclusion where it belongs: at the end of the analysis.  Only after considering all the factors and deciding that the fairer of the two options is to dismiss the action, can the court apply the label “indispensable” party.  The label is a conclusion, not a substitute for practical considerations and analysis.  This point was driven home forcefully by the Supreme Court in a major decision rendered only two years after the rewriting of Rule 19.  Provident Tradesmens Bank & Trust Co. v. Patterson, 390 U.S. 102, 88 S. Ct. 733, 19 L.Ed.2d 936 (1968).

Even though the absent person is commonly called an indispensable party, the rules reinforce that until analysis is completed, this conclusory label should not be applied.  The motion to dismiss is to be made for “failure to join a party under Rule 19.”  Fed. R. Civ. P. 12(b)(7).  A party is instructed as to waiver for failure “to join a person required by Rule 19(b).”  Fed. R. Civ. P. 12(h)(2).  Accordingly, the proper term for such a party is a “Rule 19 party.”  The term “indispensable party” is conspicuously absent from the rules.

3.  Supplemental Jurisdiction for Joinder of Parties

As discussed in I.B.1, supra, pendent and ancillary jurisdiction were devices that supported joinder of additional claims in a federal lawsuit.  Despite several attempts by lower federal courts to establish a doctrine of pendent party jurisdiction, these efforts were uniformly rejected by the Supreme Court.

     With the adoption of supplemental jurisdiction in 1990, the possibility of supplemental jurisdiction over additional parties to a lawsuit arose.  First, § 1367(a) provides “[s]uch supplemental jurisdiction shall include claims that involve the joinder or intervention of additional parties.”  Second, § 1367(b) applies only to diversity cases.

Consequently, a case in federal court on any basis save diversity alone will allow the joinder of additional, nondiverse parties.  Consider the facts of Finley v. United States, 490 U.S. 545, 109 S. Ct. 2003, 104 L.Ed.2d 593 (1989).  Plaintiff, a citizen of California, sued the United States in federal court when her husband’s airplane crashed at the San Diego airport; she sought to join a second defendant, the city of San Diego, a California citizen, on a state law tort claim arising out of the same crash.  The Supreme Court refused pendent party jurisdiction.  Today, both of these claims could be brought into federal court.  Plaintiff’s suit against the United States comes in because the United States is a party.  28 U.S.C. § 1346.  Plaintiff’s claim against the city comes in by supplemental jurisdiction.  This is so because § 1367(a) requires the two claims be part of the “same case or controversy under Article III.”  Only one airplane crashed; everything arose out of that single accident.  Further, the statute specifically covers “claims that involve the joinder or intervention of additional parties.”  That is the end of the jurisdictional analysis since § 1367(b) applies only to a “civil action of which the district courts have original jurisdiction founded solely on section 1332.”

An unintended glitch in the statute may even allow permissive joinder of plaintiffs who fail to meet the diversity requirements.  Consider, for example, two subcontractors, both citizens of state A, who wish to sue defendant contractor, a citizen of state B, for breach of their separate contracts in the same construction project.  One plaintiff seeks $400,000, but the other plaintiff seeks only $27,000.  Clearly, one plaintiff satisfies diversity requirements, but the other does not.  Can the two plaintiffs join permissively to sue together in federal court?

The question of joinder is easily answered.  Both parties sue on contracts arising from the same construction project, so the same transaction or occurrence is involved (one building project); at least one common question may arise on the events of the project or in interpretation of the standard-form contracts.

    The more interesting question is whether diversity jurisdiction between plaintiff one and defendant allows supplemental jurisdiction over plaintiff two.  We first look to § 1367(a) and determine that all parties were involved in one constitutional case or controversy (only one building project).  Then we look at § 1367(b), which applies since the sole basis for federal jurisdiction is diversity.  The statute reads “the district courts shall not have supplemental jurisdiction under subsection (a) over claims by plaintiffs against persons made parties under Rule * * * 20 * * *.”  This is a claim by a person made party under Rule 20, not a claim against a person made party under Rule 20, so § 1367(b) does not speak to this situation.  How should a court respond to this apparent drafting error?  Some federal courts read the statute as it is written and allow supplemental jurisdiction.  E.g.Stromberg Metal Works, Inc. v. Press Mechanical, Inc., 77 F.3d 928 (7th Cir. 1996).

F.  Intervention

 

A person who is not a party to a lawsuit can force his way in and become a party through the joinder device of intervention.  The outsider can intervene into the lawsuit, either as a plaintiff or as a defendant.  Should the intervention be successful, the intervenor becomes a full-fledged party to the suit.

Intervention originated in Roman law to give a nonparty a means to protect an interest when that interest might be affected by a decision in a lawsuit that the losing party chose not to appeal; it developed in different forms in the common law and equity courts.[xii]  The device was taken into practice under the codes, and a typical code required the nonparty to show she had an interest in the subject matter of the lawsuit that was not represented by the current parties.

Federal Rule 24 provides two types of intervention: of right and permissive.  When the nonparty satisfies the requirements of Rule 24(a), it has a right to intervene.  When the nonparty is unable to satisfy that rule, it still may intervene permissively under Rule 24(b) in the discretion of the court.  Of course, since the court must decide whether the requirements of Rule 24(a) are satisfied, in a large sense all intervention is permissive.

In a second sense also, all intervention is permissive.  A nonparty is never required to intervene.  It can choose to remain outside a lawsuit and attempt to protect its interests in a separate suit.  Due process prevents binding nonparties with the result of a lawsuit.

The joinder device of intervention balances a number of interests.  It prevents persons from impairing the rights of nonparties through a lawsuit.  It promotes efficiency by allowing entire controversies to be resolved in a single lawsuit.  It balances the control of the litigation by the original parties with control shared with the new party; this seems to be part of the general trend toward dilution of party control in favor of court governance of a lawsuit.

1.  Intervention of Right

Federal Rule 24(a) places three essential requirements on a party seeking to intervene of right.  First, the application for intervention must be timely.  The rule provides no guidelines of timeliness, so the court will consider the matter on an individual case basis.  The court will consider such things as the stage of the litigation, the reasons for any delay in application, and any possible prejudice to the existing parties should the intervention be allowed.  Certainly an application to intervene made during the pleading stage of the litigation will be timely; later applications raise possible delay for the original parties.

Second, the nonparty must show (a statute granting a right to intervene or) “an interest relating to the property or transaction that is the subject of the action and [the movant] is so situated that disposing of the action may as a practical matter impair or impede the movant’s ability to protect its interest.”  Fed. R. Civ. P. 24(a) (emphasis added).  A person who claims the proceeds of an insurance policy has an interest in that policy that would support intervention into a suit between another claimant for the proceeds and the insurance company.  A lienholder of a property has an interest in a lawsuit involving that property.  The interest need not always be economic.  The federal courts have recognized a broad range of interests to support intervention, including economic, environmental, and educational.[xiii]

The rule also requires that the nonparty’s interest may be impaired or impeded.  Early cases held that the nonparty had to be bound by the potential adverse decision.  This standard was almost impossible to satisfy since due process prevents binding a nonparty.  Therefore, the rule was rewritten a number of times so that today it requires only that the nonparty’s interest may be impaired or impeded as a practical matter.  For example, in a challenge by environmentalists to pollution caused by an industrial plant, potential relief might include closing the plant.  Employees, businesses in the locality, and even the county that might have its tax basis eroded would likely be held to have interests that as a practical matter might be impaired.

Third, the nonparty must show that existing parties do not adequately represent the movant’s interest.  For example, when a nonparty seeks the proceeds of an insurance policy, neither other claimants to the proceeds nor the insurance company represents the nonparty’s interest in any way.  On the other hand, assume a town zones land for open space.  The landowner sues the town for a declaratory judgment that the ordinance is invalid.  Will an environmental group that supports zoning the land for open space be entitled to intervene of right?  The answer is no because the town is already defending the zoning and adequately represents that interest.

This requirement that representation not be adequate can interplay with the timeliness requirement.  For example, assume parents of schoolchildren sue the school board for racial discrimination.  Other parents support the board policies, but they cannot intervene because the board is adequately representing their interests.  The lengthy litigation results in a judgment by the trial court that the policies are discriminatory.  The school board, for various reasons, decides not to appeal.  Can the parents supporting the board’s policies now intervene for purpose of prosecuting the appeal?  Their interests for the first time are not represented, yet post judgment in the trial court is hardly timely.  A well-known case, Smuck v. Hobson, 408 F.2d 176 (D.C. Cir. 1969), allowed intervention, but not all courts have agreed that such intervention is timely.

2.  Permissive intervention

When the nonparty cannot qualify for intervention of right, it may still seek leave of court to intervene permissively.  Federal Rule 24(b) requires only a timely application and a showing that a federal statute confers a conditional right to intervene or the movant “has a claim or defense that shares with the main action a common question of law or fact.”

As with permissive joinder of parties [see II.E.1, supra], a common question is usually easy to find.  Since the sole requirement of a common question is so minimal, much will depend on the discretion of the court.  The court will consider such things as the strength of the intervenor’s interest, possible prejudice to existing parties, and possible dilution of the control of the lawsuit by the original parties.

3.  Supplemental Jurisdiction over Intervenors

Prior to 1990, persons intervening of right generally qualified for ancillary jurisdiction while persons intervening only permissively had to establish their own independent basis for federal jurisdiction.  The advent of supplemental jurisdiction in 1990 tilted the scale strongly toward the requirement of an independent basis of federal jurisdiction for all intervenors.

As with any question of supplemental jurisdiction, we first consider § 1367(a), which requires that the claim to be added “be so related to claims in the action within such original jurisdiction that they form part of the same case or controversy under Article III.”  The court will look closely at the intervenor’s interest/claim to determine whether it is part of the same constitutional case.  Since a constitutional case is essentially a common nucleus of operative fact, supplemental jurisdiction over an added claim by a nonparty is possible.

Next, we consider § 1367(b).  When the basis for federal jurisdiction is diversity alone, this subsection applies, and it clearly eliminates any possibility of supplemental jurisdiction over intervenors:

the district courts shall not have supplemental jurisdiction under subsection (a) over claims by plaintiffs against persons made parties under Rule * * * 24 of the Federal Rules of Civil Procedure, or over claims by persons * * * seeking to intervene as plaintiffs under Rule 24 of such rules * * *.

Here the statute is clear.  It specifically eliminates supplemental jurisdiction in diversity cases over both intervening defendants and intervening plaintiffs.

G.  Interpleader

We reach the last of the joinder devices, and the last of the joinder “i”s.  Interpleader is unique, and is structurally different from all of the other joinder devices.  One of your authors likes to introduce interpleader by reading a newspaper story that appeared on the Associated Press wire several years ago:

 

The scramble is on over who gets to keep $22,350 found in a room in the Excel Inn in Bloomington [Minnesota].

The money was in $10, $20, $50 and $100 bills when [Mary Roe], a maid, found it in a brown leather briefcase while cleaning a room being rented by [John Doe] of St. Cloud [Minnesota], she said.

Roe, following the ‘finders keepers’ theory of law, has filed a claim for the cash.  Doe, who returned for the money the day after he discovered it was missing, says the money belongs to him.  He told the Bloomington police that he found it five days earlier in a paper bag near a parked car in north Minneapolis [on the day after the Super Bowl].

The situation became even more complicated when the police, who had some doubts about Doe’s story, notified the Internal Revenue Service.  The IRS now says Doe owes $33,963 in back taxes and therefore is also laying claim to the money.

In addition, Excel Inns Limited Partnership, which owns the motel, and Excel Management Associates, which operates it, have filed a claim to the money on the theory that they may have more legal right to the cash than Roe.

The city of Bloomington, which has custody of the money, [will sue] to have the ownership question decided in court.

 

What type of action will the city file?  The stakeholder (the city) is willing to hand over the stake (the briefcase of cash) to one of the claimants, but does not want to hand it over to one claimant, be sued by another for it, and have to pay twice.  The joinder device of interpleader was created for exactly this situation.

Originally, a true bill in interpleader required the stakeholder to deposit the stake into court and step back to allow the claimants to compete for it.  More recently, (an action in the nature of) interpleader allows the stakeholder also to claim the property.

Although interpleader provides a joinder device to bring all potential claimants into a single action, it does not supply personal jurisdiction over all the claimants.  Because of this weakness, the first federal interpleader act was passed in 1917.  Today, it provides a method of nationwide service on claimants.  This is known as statutory interpleader.  A second type of federal interpleader is also available under Federal Rule 22.  This is known as interpleader under the rule.  Both types of federal interpleader have differing subject matter jurisdiction, personal jurisdiction, service, and other requirements, so the stakeholder may choose one or the other depending on how the situation fits each.

1.  Statutory Interpleader

Statutory interpleader is spread through three sections of title 28: §§ 1335, 1397, and 2361.  Taken together, these sections eliminate many of the jurisdictional problems that otherwise would exist in federal court.  Only minimal diversity is required between two or more claimants to the property (§ 1335); the citizenship of a plaintiff bringing the interpleader is thus irrelevant unless the plaintiff is also a claimant (an “action in the nature of interpleader”).  A minimum jurisdictional amount of $500 is required (§ 1335).  The stakeholder must pay or place the stake into court (§ 1335).  Venue may be laid in a district where any claimant resides (§ 1397).  Process may be served nationwide (§ 2361).  The district court may enjoin claimants from pursuing the property in any other state or federal court (§ 2361).

2.  Interpleader under the Rule

Interpleader under Federal Rule 22 in many ways is less desirable than statutory interpleader, yet because of the differing jurisdictional requirements, it may be the only one of the two types of federal interpleader available to the stakeholder.

Interpleader under the rule has no special diversity jurisdiction provisions, which means that standard diversity requirements apply.  The plaintiff stakeholder must be of citizenship diverse from all defendants/claimants (§ 1332).  The stake must be of a value exceeding $75,000 (§ 1332).  Venue must be laid under standard venue rules (§ 1391).  A defendant may seek interpleader by way of a counterclaim (statutory interpleader is silent on this possibility).  The rule is silent on whether the court may enjoin claimants from proceeding against the stake in other actions.

III. QUESTIONS ON JOINDER

Instructions.  This section contains questions for you to answer to test and strengthen your knowledge of the law of joinder and supplemental jurisdiction Use your scrolling feature so that the screen shows only the question.  Answer the question yes, no, or maybe, and formulate your reasoning, then scroll down to compare your answer to the authors’ answer.  P represents plaintiff and D represents defendant.  For all questions, assume you are in federal court.

A. Counterclaims

 

Q-1.  Part 1.  P sues D for negligence in an auto accident.  D wishes to counterclaim against P for negligence in the same auto accident.  Is this a compulsory counterclaim? Part 2.  Does this counterclaim qualify for supplemental jurisdiction?

 

 

 

Answer to Q-1.  Part 1.

Yes.  A compulsory counterclaim arises out of the same transaction or occurrence as the original claim.  Fed. R. Civ. P. 13(a)(1)(A).  One auto accident is one transaction or occurrence.  This compulsory counterclaim must be asserted or lost.

Part 2.  Yes.  Since it arises out of the same transaction or occurrence, this compulsory counterclaim is part of the same case or controversy under Article III, as required by § 1367(a).  The counterclaim is asserted under Fed. R. Civ. P. 13(a); Rule 13 is not referenced in § 1367(b).  Because § 1367(a) applies, and § 1367(b) does not, the counterclaim qualifies for supplemental jurisdiction.

Supplemental jurisdiction may be unnecessary.  Since the original claim was for negligence, we must assume the basis for federal jurisdiction is diversity.  When the parties are diverse for the claim, they must also be diverse for the counterclaim.  Supplemental jurisdiction would be needed only should the amount of the counterclaim fall short of the jurisdictional amount required by § 1332.

 

 

 

 

Q-2.  Part 1.  P sues D for negligence in an auto accident.  D wishes to counterclaim against P for an antitrust violation arising out of a previous business relationship.  Is this a compulsory counterclaim?  Part 2.  Does this counterclaim qualify for supplemental jurisdiction?

 

 

 

Answer to Q-2.  Part 1.

No.  The claim is for an auto accident.  The counterclaim is for an antitrust violation arising from activities factually unrelated to the auto accident.  That means the two claims arise from different transactions or occurrences, so it is not a compulsory counterclaim.  Defendant may assert this as a permissive counterclaim.  Fed. R. Civ. P. 13(b).

Part 2.  No.  Since the permissive counterclaim does not arise from the same transaction or occurrence, it cannot be part of the same case or controversy under Article III as required by § 1367(a).

In this question, the counterclaim does not require supplemental jurisdiction.  An antitrust violation is a federal question.  It comes into federal court under § 1331.

 

 

 

 

Q-2.  Part 3.  D asserts this antitrust counterclaim against P.  P has a state tort law unfair competition claim against D arising from their previous business relationship.  Is this a compulsory counterclaim? Part 4.  Does this counterclaim qualify for supplemental jurisdiction? Part 5.  What pleading should P use to assert this unfair competition claim against D?

 

 

 

 

 

Answer to Q-2.  Part 3.

Yes, this is a compulsory counterclaim.  Once defendant asserts the antitrust counterclaim, P must assert this state tort law unfair competition claim as a counterclaim to the counterclaim since it arises out of the same transaction or occurrence as the antitrust claim.

When pleading the original complaint, P could have joined this claim as a separate count.  P and D are of diverse citizenship (or they could not be in federal court on the auto accident claim), and Fed. R. Civ. P. 18(a) allows P to assert “as many claims as it has against an opposing party.”  P chose not to join the claim at that time, but now that D has pleaded the antitrust claim, P must assert the unfair competition claim as a compulsory counterclaim or lose it.

Part 4.  As the answer to Part 3 notes, plaintiff could have joined this claim in the original complaint, so supplemental jurisdiction is not needed.  Assume, however, that the court decides P and D are not diverse.  Then the question is whether P’s state law counterclaim to D’s federal question counterclaim is covered by supplemental jurisdiction.  The answer is yes.  Both arise from the same transaction or occurrence, which means the same case or controversy under Article III.  That satisfies § 1367(a).  The counterclaim is pleaded under Fed. R. Civ. P. 13(a), which is not listed in § 1367(b).

Part 5.  Plaintiff should assert this compulsory counterclaim in the answer to the counterclaim: “Only these pleadings are allowed: * * * (3) an answer to a counterclaim designated as a counterclaim * * *.  Fed. R. Civ. P. 7(a).

 

 

 

 

Q-3.  Part 1.  P sues D for negligence in an auto accident.  D wishes to counterclaim against both P and third party X for negligence in the same auto accident.  Can D do this in a compulsory counterclaim?

Part 2.  Assume P is a citizen of New York, D is a citizen of New Jersey, and X is a citizen of New Jersey.  Does the counterclaim against both P and X qualify for supplemental jurisdiction?

 

 

 

 

 

Answer to Q-3.

Yes.  D can assert the compulsory counterclaim against P, and join an additional defending party to the counterclaim under Fed. R. Civ. P. 13(h): “Rules 19 and 20 govern the addition of a person as a party to a counterclaim or crossclaim.”  Rule 20, governing permissive joinder of parties, allows joinder when the claim against the added party arises out of the same transaction or occurrence and involves a common question of law or fact.  Those requirements are met.

Part 2.  Yes.  The compulsory counterclaim falls within § 1367(a) as part of the same case or controversy under Article III.  The statute also provides “supplemental jurisdiction shall include claims that involve the joinder or intervention of additional parties.”

The supplemental jurisdiction so provided by § 1367(a) is not destroyed by § 1367(b).  In one view, the joinder of X is pursuant to Rule 13(h), which is not mentioned at all in § 1367(b).  In another view, the joinder of X is pursuant to Rule 20, but that does not change the result because the statute refers to “claims by plaintiffs against persons made parties under Rule * * * 20 * * *.”  This is a claim by a defendant, not a plaintiff, so the statute by its terms does not apply.

 

 

 

 

 

 

Q-4.  P sells D an expensive piece of merchandise.  D falls behind in payments, and P sues.  D answers.  Some months later, D discovers the merchandise is defective.  Has D lost this compulsory counterclaim by failing to plead it in the answer?

 

 

 

 

 

Answer to Q-4.

No.  Fed. R. Civ. P. 13(a) provides several escape valves from the compulsory counterclaim rule.  The one that applies here is that defendant is required to plead “any claim that–at the time of its service–the pleader has.”  Since D was unaware of the defects at the time of answering, the claim for defects is not a compulsory counterclaim.  Other escape valves include that the potential counterclaim requires parties beyond the jurisdiction of the court and that the potential counterclaim is already the subject of another action.

 

 

 

 

 

 

Q-5.  P, a citizen of California, purchases merchandise from D, a large retail store in California.  When P fails to make payments, D engages in vigorous collection efforts.  P sues D in federal court for a violation of the Fair Debt Collection Practices Act, a federal statute.  Is D’s counterclaim for the balance due on the account a compulsory counterclaim so that it qualifies for supplemental jurisdiction?

 

 

 

 

 

 

Answer to Q-5.

Maybe.  Our answer is yes.  Rule 13(a) defines a compulsory counterclaim as arising “out of the transaction or occurrence.”  There is only one set of facts between these two parties: purchase, nonpayment, and collection efforts.  The two parties entered one contract.  These events are all tied closely together in time and space, and form a convenient trial unit.  A layperson would expect all matters arising from the purchase and payment to be tried together.  They form a common nucleus of operative fact.  These are all alternative ways of expressing that this is a single transaction or occurrence.  As such, it constitutes one constitutional case or controversy under Article III.  That means supplemental jurisdiction under § 1367(a).

Some federal courts answer no.  These courts note that the law and facts of the collection efforts are different from the law and facts of the underlying debt.  More importantly, they recognize that allowing the merchant to assert a compulsory counterclaim for the debt might discourage purchasers from suing initially to enforce the federal Fair Debt Collection Practices Act.  Viewed in that perspective, the collection efforts are not part of the same transaction or occurrence as the purchase, and thus the counterclaim is not compulsory or supplemental.  The merchant will have to sue in a separate action in state court.  See 6 Charles A. Wright, Arthur R. Miller & Mary Kay Kane, Federal Practice and Procedure: Civil § 1410 (2010) (collecting cases).

The problem with the latter view is it ignores Rule 13(a).  These two parties have engaged in only one set of interrelated dealings.  That is one transaction or occurrence.  The fact that the rule allows the merchant to bring an ordinary state law collection counterclaim in federal court, or that the result might undermine to some extent the federal policy of deterring abusive debt collection, should be irrelevant.  The remedy of the rules is an order for separate trials under Rule 42(a), not a refusal to allow the pleading.

B.  Crossclaims

Q-6.  Part 1.  P, a citizen of Florida, sues D1, a citizen of Texas, and D2, a citizen of Texas, in federal court under § 1332 for breach of contract.  Damages claimed are $200,000.  Can D1 file a counterclaim arising from the same contract against D2 for $100,000?

Part 2.  Can D2 file a crossclaim arising from the same contract against D1 for $100,000?

Part 3.  Would such a crossclaim qualify for supplemental jurisdiction?

 

 

 

 

 

Answer to Q-6.  Part 1.

No.  A counterclaim is asserted against an opposing party.  Fed. R .Civ. P. 13(a), (b).  D1 and D2 are not opposing parties as they are both on the same side of the “v.”

Part 2.  Yes.  A crossclaim is asserted against a coparty.  Fed. R. Civ. P. 13(g).  D2 and D1, being on the same side of the “v,” are co-parties.  Since the crossclaim arises from the same contract as the original claim, it clearly satisfies the requirement that it arise from the same transaction or occurrence.

Part 3.  Yes.  Since both D2 and D1 are citizens of Texas, the crossclaim does not independently satisfy the diversity requirement.  Supplemental jurisdiction will be necessary.

The short answer is that a crossclaim always qualifies for supplemental jurisdiction.  The longer answer is that the crossclaim, since it arises out of the same contract, is part of the same transaction or occurrence, and that requirement is essentially synonymous with being part of the same case or controversy under Article III as required by § 1367(a).  And Rule 13(g) is not one of the joinder rules eliminated from the operation of § 1367(a) by provision of § 1367(b).  Since § 1367(a) applies, and § 1367(b) does not, supplemental jurisdiction exists.

 

 

 

 

 

 

Q-6.  Part 4.  Can D1 then file a counterclaim against D2 for $100,000?

Part 5.  Would the counterclaim qualify for supplemental jurisdiction?

 

 

 

 

 

Answer to Q-6.  Part 4.

Yes.  Once a defendant asserts a cross-claim against another defendant, the two become opposing parties, and the proper device for the second defendant to assert a claim back against the crossclaiming defendant is a counterclaim.  Since all of these claims arise from the same contract, this would be a compulsory counterclaim.

Part 5.  Yes.  Since both D1 and D2 are citizens of Texas, the counterclaim does not meet the diversity requirement itself.  Supplemental jurisdiction will be necessary.

The short answer is that a compulsory counterclaim always qualifies for supplemental jurisdiction.  The longer answer is that the counterclaim, since it arises out of the same contract, is part of the same transaction or occurrence, which makes it so related to “claims in the action within such original jurisdiction that they form part of the same case or controversy under Article III.” So § 1367(a) applies, and Rule 13(a) is not one of the joinder rules eliminated by §1367(b).

 

 

 

 

 

 

Q-7.  Part 1.  P former employee, a citizen of Illinois, sues both D1 former employer, a corporation incorporated and operating solely within Illinois, and D2 former supervisor, a citizen of Illinois, for a violation of Title VII in firing her.  Can D2 crossclaim against D1 for indemnity to be paid in the event P succeeds in the lawsuit?

Part 2.  Would such a crossclaim qualify for supplemental jurisdiction?

 

 

 

 

 

Answer to Q-7.  Part 1.

Yes.  This crossclaim against a coparty arises out of the same transaction or occurrence as the original claim.  The crossclaim need not be mature at the time of pleading: “The crossclaim may include a claim that the coparty is or may be liable to the cross-claimant for all or part of a claim asserted in the action against the cross-claimant.”  Fed. R. Civ. P. 13(g).

Part 2.  Yes.  The crossclaim does not satisfy federal jurisdiction itself because it presents a state law question and the parties are not diverse.

Because the crossclaim is for possible indemnity in the event P wins her claim, the crossclaim arises from the same transaction or occurrence as the original claim and so is part of the same case or controversy under Article III.  That satisfies § 1367(a).  Since the original federal jurisdiction was based on a federal question under § 1331, the second paragraph of the supplemental jurisdiction statute need not be studied: § 1367(b) applies only when diversity is the sole basis for federal jurisdiction.  Supplemental jurisdiction exists.

 

 

 

 

 

 

Q-8.  P, a citizen of Arizona, sues D1, a citizen of California, and D2, a citizen of California, for negligence in an auto accident occurring in California.  P claims $100,000.  Will a crossclaim by D1 against D2 for contribution of $50,000 qualify for supplemental jurisdiction?

 

 

 

 

 

 

Answer to Q-8.

Yes.  The crossclaim does not satisfy federal jurisdiction itself because it presents a state law question and the jurisdictional amount is not sufficient.

Because the crossclaim is for possible contribution in the event P wins the claim, the crossclaim arises from the same transaction or occurrence as the original claim and so is part of the same case or controversy under Article III.  That satisfies § 1367(a).  Since the crossclaim is joined under Rule 13(g), which is not enumerated, § 1367(b) does not apply.

 

 

 

 

 

Q-9.  P sues D1 and D2 for conspiracy to violate the federal antitrust laws.  Can D1 file a crossclaim against D2 for breach of an unrelated contract?

 

 

 

 

 

Answer to Q-9.

No.  Fed. R. Civ. P. 13(g) requires that a cross-claim arise out of the same transaction or occurrence.  A factually unrelated claim by definition does not.

 

 

 

 

 

Q-10.  Part 1.  P, a citizen of Kentucky, sues D1, a citizen of Ohio, and D2, a citizen of Ohio, for breach of contract.  D1 files a cross-claim against D2 for breach of the same contract.  Can D1 at the same time join an unrelated tort claim against D2?

Part 2.  Would the unrelated tort claim qualify for supplemental jurisdiction?

 

 

 

 

 

 

Answer to Q-10.  Part 1.

Yes. Once D1 asserts a proper crossclaim against D2, he can join any other claims he has against that party.  Fed. R. Civ. P. 18.

Part 2.  No.  Diversity is absent between D1 and D2, so supplemental jurisdiction is required to bring the crossclaim and the additionally joined claim into federal court.  The crossclaim is based on the same contract and so qualifies for supplemental jurisdiction.  The unrelated claim, even though Rule 18 allows it to be joined, must find its own way into federal court.  Because the added claim does not arise out of the same transaction or occurrence, it does not satisfy the requirement of § 1367(a) that it be part of the same case or controversy under Article III.  No supplemental jurisdiction.

C.  Third-party claims

 

Q-11.  P commences an action against D for negligence arising from an auto accident.  P serves the complaint on D on June 20.  D answers with a denial on July 1.  D serves a third-party complaint on third-party defendant [3D] on July 18.  Will the court strike the third-party complaint on motion of either P or 3D?

 

 

 

 

 

Answer to Q-11.

Yes.  Rule 14 allows defendant 14 days after service of the answer to serve a third-party complaint without leave of court.  Since more than 14 days have passed, defendant will have to obtain leave of court.  This early in the litigation, the court is highly likely to grant leave to assert the impleader, so an answer of no (on reasoning that the impleader is not timely but that the court will likely grant leave instead of granting the motion to strike) is acceptable.

 

 

 

 

 

Q-12.  P commences an action against D for negligence arising from a three-car auto accident.  D serves a third-party complaint on 3D, the driver of the third car.  The third-party complaint asserts D was not negligent and 3D was negligent.  Will the court strike the third-party complaint on motion of either P or 3D?

 

 

 

 

 

 

Answer to Q-12.

Yes.  A third-party claim must assert derivative liability: “is or may be liable to [the third-party plaintiff] for all or part of the claim against it.”  Fed. R. Civ. P. 14(a)(1).  This is not a claim of derivative liability; it is a defense that another, alternative defendant is liable directly to the plaintiff.  It is not a proper use of third-party practice.

 

 

 

 

 

Q-13.  Part 1.  P, a citizen of Washington, sues D, a citizen of Oregon, for $250,000 for breach of contract.  Can D implead 3D, a citizen of Washington, for indemnity on liability on the contract?

Part 2.  Can 3D then assert a claim against P for $125,000 for breach of the same contract?

Part 3.  Can P then assert a claim against 3D for $125,000 for breach of the same contract?

 

 

 

 

 

Answer to Q-13.  Part 1.

Yes.  This is a proper use of third-party practice, since the liability would be derivative.  With regard to federal jurisdiction, the original claim satisfied diversity jurisdiction (Washington v. Oregon for $250,000), and the third-party claim independently satisfied diversity jurisdiction (Oregon v. Washington for $250,000), so supplemental jurisdiction need not be considered.

Part 2.  Yes.  This claim by the third-party defendant directly against the plaintiff is allowed by Fed. R. Civ. P. 14(a)(2)(D) since it arises from the same transaction or occurrence–here the same contract–as the original claim.  With regard to federal jurisdiction, the claim does not independently satisfy diversity (Washington v. Washington), so we must consider supplemental jurisdiction.  Under § 1367(a), the derivative claim is part of the same transaction or occurrence, so it is part of the same case or controversy under Article III; under § 1367(b), this is not a claim by a plaintiff against a person made party under Rule 14.  Consequently, supplemental jurisdiction exists.

Part 3.  No.  This claim by the plaintiff directly against the third-party defendant is allowed by Fed. R. Civ. P. 14(a)(3) since it arises from the same transaction or occurrence–here the same contract–as the original claim.  With regard to federal jurisdiction, however, the claim does not independently satisfy diversity (Washington v. Washington), so we must consider supplemental jurisdiction.  Even though the claim is part of the same case or controversy under Article III for § 1367(a), this is a claim by a plaintiff against a person made party under Rule 14, which is disallowed by § 1367(b).  Consequently, supplemental jurisdiction does not exist.

 

 

 

 

 

Q-14.  Part 1.  P, a citizen of Michigan, sues D, a citizen of Ohio, for an auto accident.  D asserts a permissive counterclaim against P for breach of an unrelated contract.  Assume the jurisdictional amounts in both claims are adequate.  Can P implead X, a citizen of Ohio, for indemnity on any liability on the contract?

Part 2.  Can D then assert a claim directly against X for breach of the same contract, and will the federal court have supplemental jurisdiction over that claim?

 

 

 

 

 

 

Answer to Q-14.  Part 1.

Yes.  The permissive counterclaim satisfies federal jurisdiction independently (Ohio v. Michigan for sufficient amount).  At that point, the original plaintiff, as “defending party,” may assert a third-party claim.  Fed. R. Civ. P. 14(a)(1), (b).  The third-party claim independently satisfies federal jurisdictional requirements (Michigan v. Ohio for complete indemnity), so supplemental jurisdiction is not needed.

Part 2.  Yes and yes.  The claim by the original defendant D directly against X is allowed by Fed. R. Civ. P. 14(a)(3).  With regard to federal jurisdiction, the claim does not independently satisfy diversity (Michigan v. Michigan), so we must consider supplemental jurisdiction.  The third-party claim is part of the same transaction or occurrence as the counterclaim, which is in federal court on its own independent jurisdictional basis, so it satisfies § 1367(a).  Under 1367(b), this is not a claim by a plaintiff against a person made party under Rule 14.  Consequently, supplemental jurisdiction exists.  Even though this claim by “defendant” directly against X is equivalent to a claim by an original plaintiff directly against a third-party defendant, the plain language of § 1367(b) does not cover it.  Glitch in the statute or intended, this third-party claim is in federal court.

D.  Joinder of Parties

 

Q-15.  A fire negligently started by D1 destroys P’s house.  Several months later, D2, D1’s insurance company, refuses to pay the claim without any good faith basis for refusal.  Can P permissively join both D1 and D2 as defendants in a single lawsuit?

 

 

 

 

Answer to Q-15.

Yes.  The first requirement for permissive joinder of defendants, that the claims against both arise from the same transaction or occurrence, or series of transactions or occurrences, is satisfied.  Even though the claim against D1 and the claim against D2 are separated in time by several months, will involve little overlap in evidence, and are on different theories of recovery, the important fact is that there was only one fire.  Everything arose from that single event.  That is properly considered a single “transaction,” and without question both claims are from a related “series of transactions or occurrences.”  Fed. R. Civ. P. 20(a)(2)(A).

The second requirement, that at least one common question will arise [Fed. R. Civ. P. 20(a)(2)(B)], is easily met.  Some of the common questions are D1’s negligence in the fire, the amount of P’s damages, etc.

 

 

 

 

 

Q-16.  Part 1.  P1 is a pedestrian on the sidewalk.  A car driven by D runs a stop sign and broadsides a car driven by P2.  P2’s car skids onto the sidewalk and injures P1.  Can P1 and P2 permissively join together to sue D in a single lawsuit?

Part 2.  Assume P1 and P2 have not yet sued D.  More than a year later, P1 still has not recovered from the accident injuries.  She consults D2, a physician, and is treated negligently.  Can P1 and P2 permissively join together as plaintiffs to sue in a single lawsuit and permissively join both D and D2 as defendants?

 

 

 

 

 

 

Answer to Q-16.  Part 1.

Yes.  The first requirement for permissive joinder of plaintiffs, that the claims of both arise from the same transaction or occurrence, is satisfied.  There was only one accident.  The second requirement of a single common question is easily met.  Issues of D’s negligence (breach of duty, causation) will arise with regard to both plaintiffs.  Fed.R.Civ.P. 20(a)(1)(A),(B).

Part 2.  Yes.  Both P1 and P2 can permissively join as plaintiffs for the reasons given in the answer to Part 1.  Plaintiffs can permissively join both D and D2 as defendants for the same reasons given in the answer to Q-15.  Even though the medical malpractice occurred more than a year after the accident, it clearly arose from the events of the accident and therefore is part of the same series of transactions or occurrences.  P1 would not have been in the physician’s office but for the accident injuries.  The extent of P’s injuries is one apparent common question.

 

 

 

 

 

Q-17.  P, a citizen of Louisiana, undergoes an operation in which a plate and screw device is implanted into his back.  The device breaks.  P sues D device manufacturer, a citizen of Pennsylvania, in federal court.  P sues D surgeon, a citizen of Louisiana, in a separate suit in state court.  D manufacturer moves to dismiss the federal suit for failure to join a Rule 19 party.  Should the motion to dismiss be granted?

 

 

 

 

 

 

Answer to Q-17.

No.  Joint tortfeasors are never Rule 19/indispensable parties.  The common law has always given plaintiff the option to sue one or more at his option, and the unlucky chosen defendant cannot force joinder of, or demand dismissal for failure to join, the other(s).  Temple v. Synthes Corp., 498 U.S. 5, 111 S. Ct. 315, 112 L.Ed.2d 263 (1990).

 

 

 

 

 

Q-18.  P, a citizen of Virginia, enters a contract with D1, a citizen of North Carolina, and at the same time enters a related contract with D2, a citizen of Virginia.  Both contracts refer to each other, and interpretation of one will require interpretation of the other.  Both defendants fail to perform.  P sues D1 for breach of contract.  D1 moves to dismiss for failure to join a Rule 19 party (D2 cannot be joined since diversity would be destroyed).  Should the motion be granted?

 

 

 

 

 

Answer to Q-18.

Maybe.  We hope you recognized this is a trick question.  The proper answer is that no answer can be given until we grind our way through the considerations of Federal Rule 19(a) and 19(b).  A categorical answer to this question, based only on the information so far at hand, would be to place a conclusory label on D2, which is exactly what Fed. R. Civ. P. 19 was rewritten in 1966 to avoid.

 

 

 

 

 

Q-19.  Part 1.  P, a citizen of Florida, is employed by D Corporation, incorporated in Delaware with principal place of business in Georgia.  D supervisor, a citizen of Florida, fires D “because we don’t want anyone of your religion working for us.”  Can P join both defendants in a single suit in federal court alleging two counts: 1) violation of Title VII of the Civil Rights Act of 1964, and 2) breach of the contract of employment?

Part 2.  Will the federal court have supplemental jurisdiction under § 1367 over the state law count two and D supervisor?

 

 

 

 

 

 

Answer to Q-19.  Part 1.

Yes.  Plaintiff was fired once.  The claims against both defendants arise from the same transaction or occurrence.  Common questions not only exist, but also they likely predominate.

Part 2.  Yes.  Supplemental jurisdiction exists under § 1367(a) because this is a single set of facts arising from P’s termination: it is one case or controversy under Article III.  The analysis need not continue to § 1367(b) because that subsection applies only when the sole basis of jurisdiction is diversity.  Here, a federal question–alleged violation of Title VII, a federal statute–is presented.  Supplemental jurisdiction under § 1367(a) includes “joinder * * * of additional parties.”

 

 

 

 

 

Q-20.  Part 1.  P1, P2, and P3, all citizens of Pennsylvania, together open a small business and rent a property from D in New Jersey.  The business does not succeed, and Ps believe the failure is because D fraudulently represented the nature of the property.  P1 loses $100,000, P2 loses $50,000, and  P3 loses $5,000.  Can the three plaintiffs join together to sue D?

Part 2.  Will the court have supplemental jurisdiction over P2 and P3?

Part 3.  D counterclaims against P1 for rent of $5000 still due on the property.  Will the court have supplemental jurisdiction over cross-claims by P1 against the other two plaintiffs for contribution?

 

 

 

 

 

Answer to Q-20.  Part 1.

Yes.  This is permissive joinder of plaintiffs.  All three plaintiffs were involved in the same business deal for the same property.  This is the same transaction or occurrence.  Common questions involving all four include whether a fraudulent representation was made, whether defendant acted with scienter, and the like.  Fed. R. Civ. P. 20(a)(1).

Part 2.  Maybe.  The answer to this question is the same as was discussed above in II.E.3.  Diversity exists between P1 and D.  The amounts claimed by P2 and P3 are insufficient.  We first look to § 1367(a) and determine that all parties were involved in one constitutional case or controversy (only one business deal involving one property), and the statute allows “joinder * * * of additional parties.”  Then we look at § 1367(b): “The district courts shall not have supplemental jurisdiction under subsection (a) over claims by plaintiffs against persons made parties under Rule * * * 20 * * *.”  This is a claim by persons made parties under Rule 20, not a claim against persons made parties under Rule 20, so the plain language of § 1367(b) does not cover this situation.  How should a court respond to this apparent drafting error?  Some federal courts read the statute as it is written and allow supplemental jurisdiction even though the rather clear intent was not to allow supplemental jurisdiction in such a situation.

Part 3.  Maybe.  The federal court certainly has supplemental jurisdiction over the compulsory counterclaim.  The question is whether it has supplemental jurisdiction over P1’s cross-claims against P2 and P3.  The cross-claims for contribution arose from the single business deal, so § 1367(a) provides jurisdiction.  While the other plaintiffs are in the position of defending parties to the cross-claims, they are still “plaintiffs” in the lawsuit.  The relevant sentence of § 1367(b) again is “The district courts shall not have supplemental jurisdiction under subsection (a) over claims by plaintiffs against persons made parties under Rule * * * 20 * * *.”  Since the other three plaintiffs were originally made parties under Rule 20, the plain language reading of the statute is no supplemental jurisdiction.  Another glitch.

E.  Intervention

 

Q-21.  When a man is killed in an auto accident, his two children as next of kin bring a wrongful death action against the other driver for negligence.  Upon learning of the action, a woman who claims she is the illegitimate daughter of the deceased seeks to intervene as a plaintiff to share in any recovery.  Should this nonparty be allowed to intervene of right?

 

 

 

 

Answer to Q-21.

Yes.  The intervenor has an interest in the damages for wrongful death that may be recovered.  The intervenor’s interest may be impaired: the two existing plaintiffs may obtain a recovery that exhausts defendant’s resources.  The intervenor’s interest is not adequately represented: the two plaintiffs have no interest in cutting her in for a share and the defendant has no interest in providing any share to cut.  See Fed. R. Civ. P. 24(a)(2).  So long as the petition for intervention is timely, it should be granted.

 

 

 

 

Q-22.  Part 1.  A corporation wishes to open a drug treatment facility in a vacant building that formerly was a neighborhood school.  To do so, it must obtain a conditional use permit from the city.  Under pressure from a neighborhood association comprised of neighboring property owners, the city council turns down the permit.  The corporation sues the city to require it to issue the permit.  Will the neighborhood association be allowed to intervene of right as a defendant?

Part 2.  Will the neighborhood association be allowed to intervene permissively as a defendant?

Part 3.  Assume the association does not seek to intervene.  When the trial court orders the city to issue the conditional use permit and the city council decides not to appeal, will the association be allowed to intervene of right to pursue the appeal?

 

 

 

 

 

Answer to Q-22.  Part 1.

No.  The interests of the neighborhood association are adequately represented by the city, which is defending the action.  Even though the members of the association have property interests that may be impaired, they cannot intervene of right.  See Fed. R. Civ. P. 24(a)(2).

Part 2.  Maybe.  The sole requirement that the association’s defense have a question of law or fact in common with the existing parties is easily met.  See Fed. R. Civ. P. 24(b)(1)(B).  At that point, the court will consider other factors including delay of the proceedings, control of the litigation, and value of the association’s participation.

Part 3.  Maybe.  Since the association is no longer adequately represented, it has a right to intervene, so long as it petitions for intervention in a timely fashion.  Plaintiff will argue intervention following final judgment in the trial court is not timely.  The association will argue it acted promptly as soon as it had a right to intervene.  While the majority opinion would be that the association may intervene of right [see II.F.1, supra], the court in this particular case decided the petition to intervene was untimely.  Omegon, Inc. v. City of Minnetonka, 346 N.W.2d 684 (Minn. App. 1984).

 

 

 

 

 

Q-23.  P, a citizen of Tennessee, sues D, a corporation incorporated in Nevada with principal place of business in Kentucky, for pollution of a body of water on which P is a landowner.  The basis of federal jurisdiction is diversity.  I, a citizen of Kentucky, who owns land that borders on the same body of water, petitions to intervene of right, or in the alternative permissively, as a plaintiff.  Assuming Fed. R. Civ. P. 24 allows either type of intervention, will the court have supplemental jurisdiction?

 

 

 

 

 

Answer to Q-23.

No.  Even assuming arguendo that the proposed intervention satisfies § 1367(a) since it involves the same pollution of the same body of water, § 1367(b) disallows supplemental jurisdiction in diversity cases for both intervening plaintiffs (as here) and intervening defendants.

F.  Interpleader

 

Q-24.  An insurance company, incorporated and with principal place of business in Ohio, issues a policy of life insurance with the face value of $50,000.  The insured dies.  The daughter of the insured, a citizen of California, sues in state court to recover the proceeds.  The widow of the insured, a citizen of California, informs the insurance company she also intends to claim the proceeds.  Will the insurance company be able to interplead the daughter and widow in federal court?

 

 

 

 

 

Answer to Q-24.

No.  Neither type of interpleader–statutory or under the rule–is available on these facts.  Statutory interpleader requires minimal diversity of claimants [see 28 U.S.C. § 1335]; both claimants are from California.  The citizenship of the insurance company is irrelevant since it is not claiming the stake.  Interpleader under the rule [Fed. R. Civ. P. 22] is subject to standard jurisdiction requirements.  Diversity is satisfied since the insurance company is a citizen of Ohio and both claimants are citizens of California, but the jurisdictional amount is insufficient.  The insurance company will have to interplead the claimants in state court.

 

 

 

 

 

Q-25.  Part 1.  P, a citizen of Illinois, advertises a baseball autographed by Babe Ruth on an internet auction website.  P agrees to sell the ball to A, a citizen of Illinois, for $10,000.  After the auction is closed, D, a citizen of Illinois, offers P $20,000 for the ball, and P accepts.  P then receives a letter from the New York Yankees baseball club informing him that the ball was stolen from a memorabilia display owned by the club.  Will P be able to bring an action against the three claimants for federal statutory interpleader?

Part 2.  Will P be able to bring an action against the three claimants for interpleader under the rule?

 

 

 

 

 

Answer to Q-25.  Part 1.

Yes.  Minimal diversity amongst the claimants exists (Illinois and New York) and the amount in controversy is $500 or more [see 28 U.S.C. § 1335].  P may lay venue in either Illinois or New York, where the claimants reside [see §1397].  Process is available nationwide [see § 2361], so if P commences the action in Illinois, process may be served on the Yankees ball club in New York.

Part 2.  No.  Standard jurisdiction requirements apply to interpleader under the rule [Fed. R. Civ. P. 22].  Diversity jurisdiction does not exist.  Plaintiff and two defendants/claimants are citizens of Illinois.  Also, the amount in controversy does not exceed $75,000.

 

 

 

 

 

Q-26.  Part 1.  A bank incorporated and with principal place of business in New Mexico receives a deposit for $250,000 from a depositor in Texas.  The depositor dies.  The depositor’s wife, a citizen of Texas, claims the account.  Depositor’s two children, both citizens of Texas, claim the account.  Depositor’s business partner, a citizen of Texas, claims the account.  Will the bank be able to bring an action against the four claimants for federal statutory interpleader?

Part 2.  Will the bank be able to bring an action against the four claimants for interpleader under the rule?

 

 

 

 

 

 

Answer to Q-26.  Part 1.

No.  Even though § 1335 requires only minimal diversity, it requires diversity amongst the claimants, not between the stakeholder and the claimants.  All claimants here are citizens of Texas.  Statutory interpleader is not available.

Part 2.  Yes.  Complete diversity exists between the plaintiff bank and all defendant claimants.  The amount in controversy exceeds $75,000.  The venue statute [28 U.S.C. § 1391(a)] allows the bank to lay venue in Texas (where all defendants reside) or New Mexico (where a substantial part of the property is located).  Should the bank commence the action in federal court in New Mexico, personal jurisdiction over the Texas defendants may be a problem.

IV. COMPUTER EXERCISES

You are now ready for additional work on joinder and supplemental jurisdiction in the computer‑assisted exercises available through CALI.  We have not written a computer-assisted exercise to accompany this written exercise on joinder and supplemental jurisdiction, but the CALI library includes three self-contained exercises from which to choose.  All three exercises are written by David Welkowitz, Professor of Law, Whittier Law School.

 

A. CALI CIV 11: A Review of Joinder Concepts

This exercise is intended to allow students to review joinder of claims and parties under the Federal Rules. The exercise uses a construction project litigation as the basis for the questions. The litigation grows gradually, adding claims and parties along the way. At each step, the student is asked questions about the propriety of joining the claim and/or the party.

 

B.  CALI CIV 18: Joinder of Claims and Parties

This exercise is designed to be used in different ways. Students may use it as a tutorial to accompany assigned readings, as a supplement to reinforce concepts discussed in class, or as a review before exams. The program is interactive, requiring the student to respond to various questions and hypotheticals to learn the principles embodied in the rules. It does not assume any specific knowledge of the joinder rules–it is designed to teach the rules from scratch.

The program uses hypertext links between various parts of the program. These links offer students options in navigating through the program so they are not forced to follow a particular order. The user is the master of the organization. All of the rules and statutes that are needed are available as part of the program and may be viewed at any time by selecting an on-screen button.

The program includes units on a variety of joinder topics: claim joinder (Rule 18); permissive party joinder (Rule 20); counterclaims (Rule 13); crossclaims (Rule 13); third-party claims (Rule 14); compulsory joinder of parties (Rule 19); and intervention (Rule 24). It also contains an extensive unit devoted to the subject matter jurisdiction problems raised by these rules. Finally, there is a review unit to allow the user to apply the principles learned in the lesson.

 

C.  CALI CIV 21: An Interpleader Primer

This exercise briefly describes the concept of interpleader and some of the historical limitations on the remedy, but its focus is on statutory interpleader [28 U.S.C. § 1335] and interpleader under the rule [Fed. R. Civ. P. 22]. The lesson introduces the various procedural issues involved–such as subject matter jurisdiction, personal jurisdiction, and venue–and highlights the differences between statutory and rule interpleader on these subjects. The lesson also contains a segment on the problem presented in enjoining other pending actions. The lesson requires the student to use the relevant statutes and rules, which are included in the lesson.



[i]
The typical pattern of a code was to list categories of causes of action that could be joined:

(1) contracts, express or implied; (2) injuries to the person; (3) injuries to the character; (4) injuries to the property; (5) actions to recover real property with or without damages; (6) actions to recover chattels with or without damages; (7) claims against a trustee by virtue of a contract or operation of law; (8) actions arising out of the same transaction or transactions connected with the same subject of action.

See Charles E. Clark, Code Pleading 441 (2d ed. 1947).(Return to text)

[ii]This exercise does not consider certain topics related to joinder: real party in interest (action must be brought in the name of the person who will benefit from a recovery), and capacity to sue or be sued (ability of a person to represent her own interests in an action).  See 6A Charles A. Wright, Arthur R. Miller & Mary Kay Kane, Federal Practice and Procedure: Civil §§ 1542-73 (2010).  This exercise does not consider standing to sue (ability of a person to challenge governmental action).  See 13A Charles A. Wright, Arthur R. Miller & Edward H. Cooper, Federal Practice and Procedure: Jurisdiction §§ 3531 et seq. (2005).

This exercise also does not consider class actions, a joinder device that allows a large number of persons to join as plaintiffs (or defendants) in a single litigation.  The multiple nuances of that topic far exceed the scope of this brief treatment of joinder devices.  See generally 7A-7B Charles A. Wright, Arthur R. Miller & Mary Kay Kane, Federal Practice and Procedure: Civil 2d §§ 1751-1820 (2005). (Return to text)

[iii]The doctrine of supplemental jurisdiction applies only in federal courts, which are courts of limited subject matter jurisdiction.  State courts of general jurisdiction do not have problems with limited subject matter jurisdictional reach, so joinder problems in state courts are limited to the joinder devices. (Return to text)

[iv]United Mine Workers v. Gibbs, 383 U.S. 715, 725, 86 S. Ct. 1130, 1138, 16 L.Ed.2d 218, 227-28 (1966).  The idea that federal courts have power to hear all aspects of a case traces back to Osborn v. Bank of the United States, 22 U.S. 738, 823 (1824):

[W]hen a question to which the judicial power of the Union is extended by the constitution, forms an ingredient of the original cause, it is in the power of congress to give the circuit courts jurisdiction of that cause, although other questions of fact or of law be involved in it.

While the Gibbs opinion referred to the federal claim and the state law claim, the better terminology would be to refer to two theories of recovery within the same claim, since a claim is commonly recognized as all facts comprising a transaction or occurrence, which would be roughly synonymous with the “common nucleus of operative fact.”  Gibbs meshes nicely with the scope of the federal claim for relief.  See Exercise Three, part I.C.2(Return to text)

[v]Finley v. United States, 490 U.S. 545, 109 S. Ct. 2003, 104 L.Ed.2d 593 (1989); Owen Equip. & Erection Co. v. Kroger, 437 U.S. 365, 98 S. Ct. 2396, 57 L.Ed.2d 274 (1978); Aldinger v. Howard, 427 U.S. 1, 96 S. Ct. 2413, 49 L.Ed.2d 276 (1976). (Return to text)

[vi]28 U.S.C. § 1367(a)-(b).  Paragraph (c) outlines situations in which the federal court may decline to exercise supplemental jurisdiction.  Paragraph (d) is a saving statute for statute of limitations purposes should the court send a state claim back to state court.  Paragraph (e) defines state for purposes of the statute. (Return to text)

[vii]Fed. R. Civ. P. 13(a)(1) provides as follows:

A pleading must state as a counterclaim any claim that–at the time of its service–the pleader has against an opposing party if the claim:

(A) arises out of the transaction or occurrence that is the subject matter of the opposing party’s claim; and

(B) does not require adding another party over whom the court cannot acquire jurisdiction.

As can be seen, the pleader need not state a counterclaim it does not have at the time of serving the pleading.  The rule also contains other escape valves, including that the potential counterclaim is already “the subject of another pending action.”  Fed. R. Civ. P. 13(a)(2)A).  See generally Jack H. Friedenthal, Mary Kay Kane & Arthur R. Miller, Civil Procedure § 6.7, at 361 (4th ed. 2005). (Return to text)

[viii]Claim preclusion covers all parts of the plaintiff’s claim that were or should have been adjudicated.  Most courts today accept the transactional definition of claim found in Restatement (Second) of Judgments § 24 (1982): “the claim extinguished includes all rights of the plaintiff to remedies against the defendant with respect to all or any part of the transaction, or series of connected transactions, out of which the action arose.” (Return to text)

[ix]See 6 Charles A. Wright, Arthur R. Miller & Mary Kay Kane, Federal Practice and Procedure: Civil 2d § 1417 (2010). (Return to text)

[x]Fed. R. Civ. P. 20(a) reads in relevant part as follows:

(1) Plaintiffs. Persons may join in one action as plaintiffs if:

(A) they assert any right to relief jointly, severally, or in the alternative with respect to or arising out of the same transaction, occurrence, or series of transactions or occurrences; and

(B) any question of law or fact common to all plaintiffs will arise in the action.

      (2) Defendants.  Persons * * * may be joined in one action as defendants if:

(A) any right to relief is asserted against them jointly, severally, or in the alternative with respect to or arising out of the same transaction, occurrence, or series of transactions or occurrences; and

(B) any question of law or fact common to all defendants will arise in the action. * * * (Return to text)

[xi]7 Charles A. Wright, Arthur R. Miller & Mary Kay Kane, Federal Practice and Procedure: Civil 3d §§ 1601-04 (2001). (Return to text)

[xii]See Jack H. Friedenthal, Mary Kay Kane & Arthur R. Miller, Civil Procedure § 6.10 (4th ed. 2005). (Return to text)

[xiii]See 7C Charles A. Wright, Arthur R. Miller & Mary Kay Kane, Federal Practice and Procedure: Civil 3d § 1908 (2007). (Return to text)

License

Icon for the Creative Commons Attribution-NonCommercial-ShareAlike 4.0 International License

Computer-Aided Exercises in Civil Procedure by Center for Computer-Assisted Legal Instruction is licensed under a Creative Commons Attribution-NonCommercial-ShareAlike 4.0 International License, except where otherwise noted.